Master the fundamental mean value theorems that connect function values to derivatives, enabling powerful applications in analysis and inequality proofs.
A point is a local maximum of if there exists such that for all . Local minimum is defined similarly with .
If has a local extremum at and exists, then:
Suppose is a local maximum. For small: .
For small: .
Since exists, both one-sided limits equal , so .
Points where are called critical points or stationary points.
Important Distinction:
Problem: Find all critical points of .
Solution:
Setting :
Critical points:
Analysis: (local min), (local max), (local min)
Example: Consider .
has a local minimum at , but does not exist!
This shows that differentiability is essential in Fermat's theorem. Extrema can occur at points where the derivative doesn't exist.
To find all extrema of on :
Problem: Find the global max and min of on .
Solution:
Step 1: Find critical points.
Critical points: .
Step 2: Evaluate at critical points and endpoints.
Global max: 10 at . Global min: -22 at .
At a critical point (where ):
If and , then there exists such that:
By the Extreme Value Theorem, attains max and min on .
Case 1: If , then is constant, so for all .
Case 2: If , since , at least one of is attained at some . By Fermat's theorem, .
If a smooth curve starts and ends at the same height, there must be a point where the tangent is horizontal.
Conditions Check:
Problem: Show that has at least one root in .
Solution:
Check: , .
Since and is continuous, by IVT, there exists with .
Note: This uses the Intermediate Value Theorem, not Rolle's theorem directly!
Problem: Show that has exactly one real root.
Solution:
Existence: , . By IVT, at least one root exists.
Uniqueness: Suppose two roots exist. Then .
By Rolle's theorem, : .
But for all . Contradiction!
Therefore, has exactly one real root.
Problem: If is a polynomial with distinct real roots, how many roots does have?
Solution:
Let the roots be .
Between consecutive roots and , apply Rolle's theorem:
, so with .
Conclusion: has at least real roots.
If has points where takes the same value (i.e., ), then has at least zeros, has at least zeros, etc.
Problem: Show that has infinitely many critical points.
Solution:
for all integers .
By Rolle's theorem, between consecutive zeros at and :
Since there are infinitely many such intervals, has infinitely many critical points.
Problem: If and , show with .
Solution:
By MVT on : with .
Consider .
and .
By Rolle's theorem: with , i.e., .
Take or : .
Counterexample when hypotheses fail:
Consider on .
There is no with ! Rolle's theorem doesn't apply.
If , then there exists such that:
Define . Then and .
By Rolle's theorem, : .
If and for all , then is constant on .
Prove: For :
Apply MVT to on (assume ):
Since : , giving .
There exists a point where the tangent line is parallel to the secant line connecting and .
Secant line slope:
Tangent line slope at ξ:
MVT says these are equal for some !
Let . Then:
for all ⟹ is strictly increasing on
for all ⟹ is strictly decreasing on
For in , by MVT:
Since and , we get .
Problem: Prove that for all .
Solution:
Apply MVT to on :
So .
Since and (with equality only at ):
Problem: If for all , prove .
Solution:
By MVT, for any :
Taking absolute values:
This is called the Lipschitz condition with Lipschitz constant .
Problem: Prove that for all .
Solution:
Case 1: . Apply MVT to on :
Since for : , so .
Case 2: . Apply MVT on :
Since for : , so .
Equality holds only when .
Lagrange's MVT is used to:
Problem: Prove that for : .
Solution:
Apply MVT to on :
Since : , so .
Connection to Numerical Methods:
MVT says that for some :
This motivates the finite difference approximation:
The error in this approximation is related to and by Taylor's theorem.
If is differentiable on and for all , then is uniformly continuous on .
Problem: If for all , show has at most one fixed point.
Solution:
Suppose and with .
By MVT:
This gives .
But . Contradiction!
If and , then :
Note (by Rolle's theorem applied to ). Define:
Then . By Rolle's theorem, : .
Cauchy's MVT is crucial for proving L'Hôpital's Rule. It generalizes Lagrange's MVT to ratios of functions.
Relationship to Lagrange's MVT:
If we set , then and:
This is exactly Lagrange's MVT!
Problem: Using Cauchy's MVT, show that for :
Solution:
Apply Cauchy's MVT to and on :
Since :
Therefore:
Preview: Cauchy's MVT is the foundation of L'Hôpital's Rule.
For the limit with :
for some between and .
As , , suggesting:
If we think of as a parametric curve, Cauchy's MVT says:
There's a point where the tangent to the curve is parallel to the chord from to .
Problem: Show that for : .
Solution:
By MVT applied to on :
We need to show (i.e., is in the right half of ).
Since is convex (), the average value of is achieved to the right of the midpoint.
Problem: Using Cauchy's MVT, show for :
Solution:
Apply Cauchy's MVT to , on :
Since :
If are differentiable with and is monotonic, then:
If is differentiable on , then takes every value between and .
Significance: Even though may not be continuous, it still has the intermediate value property. This means derivatives cannot have "jump discontinuities"—any discontinuity must be of the oscillating type.
Let be differentiable on with .
Define . Then:
Since , is decreasing near , so attains its minimum at some .
By Fermat's theorem: , so .
Darboux's theorem places strong restrictions on what functions can be derivatives:
Cannot be a derivative:
Step functions, signum function, any function with jump discontinuities
Can be a derivative:
Continuous functions, functions with only removable or oscillating discontinuities
Problem: Show that is not the derivative of any function.
Solution:
Suppose for some .
and .
By Darboux's theorem, must take all values between and .
In particular, for some .
But for all , never . Contradiction!
Example: The function (with ) has a discontinuous derivative:
For :
At : (by limit definition).
is discontinuous at 0 (oscillates), but by Darboux's theorem, it still has the IVP!
Advanced fact: If is differentiable, then is a Baire class 1 function.
This means is the pointwise limit of continuous functions. The set of discontinuities of must be "small" in a precise topological sense.
Problem: Can be expressed as for some ?
Solution:
No! By Darboux's theorem, if , then must have the IVP.
and , but never takes the value .
This violates the IVP, so cannot be a derivative.
Allowed:
Not allowed:
If is continuous on , differentiable on , and , then .
Problem: Find for ().
Solution:
By definition:
by the squeeze theorem since .
The mean value theorems form a hierarchy, each generalizing the previous:
Fermat's Theorem
Local extremum ⟹ derivative = 0
Rolle's Theorem
f(a) = f(b) ⟹ f'(ξ) = 0
Lagrange's MVT
f'(ξ) = [f(b) - f(a)]/(b - a)
Cauchy's MVT
f'(ξ)/g'(ξ) = [f(b) - f(a)]/[g(b) - g(a)]
Problem: If and , prove : .
Sketch of Solution:
Problem: Prove that for : .
Solution:
Apply MVT to on :
Since :
Multiplying by :
Problem: If for all , what can we conclude?
Solution:
Let . Then for all .
By the constant function criterion (Corollary of MVT):
Conclusion: for some constant .
If a car travels from A to B, its instantaneous speed must equal its average speed at some moment.
Example: Drive 100 km in 2 hours (average 50 km/h).
By MVT, at some instant, the speedometer showed exactly 50 km/h!
Problem: Prove that for .
Solution:
The right inequality is immediate for .
For the left: let .
Therefore for , giving .
If and exists on , with for distinct points , then:
Problem: If (f is convex), prove:
Solution Sketch:
Let . Apply MVT on and :
with .
Since , is increasing, so .
Adding and using gives the result.
Real-World Application:
A car enters a highway at 2:00 PM at marker A and exits at marker B (10 km away) at 2:06 PM. Speed limit is 100 km/h. Can police prove speeding?
Solution:
Average speed = km/h.
By MVT, at some moment the instantaneous speed equaled 100 km/h exactly.
If the car was stationary at any point, it must have exceeded 100 km/h at some other moment!
The Mean Value Theorem for Integrals is a related result:
If is continuous on , then :
This is the integral analog of Lagrange's MVT, connecting average values to function values.
Problem: Prove that for : (with equality at ).
Solution:
Let . Then .
at ; for ; for .
So has a global minimum at : .
Therefore for all , i.e., .
This module covered the fundamental mean value theorems of differential calculus:
There exists a point where the tangent line is parallel to the secant line connecting (a, f(a)) and (b, f(b)).
Rolle's theorem is a special case of Lagrange's MVT when f(a) = f(b), making the secant line horizontal.
It shows that derivatives have the intermediate value property even without being continuous. This is a strong constraint on what functions can be derivatives.
Identify the function and interval, verify hypotheses (continuity, differentiability), apply the theorem to get a point ξ with the desired property.
Local extrema are max/min in a neighborhood; global extrema are the largest/smallest values on the entire domain. Fermat's theorem applies to local extrema.
No. If f(a) ≠ f(b), use Lagrange's MVT instead. Rolle's theorem specifically requires equal endpoint values.
By Darboux's theorem, derivatives must have the IVP. Step functions have jump discontinuities, violating this property.
Apply MVT to the relevant function on an appropriate interval. The existence of ξ with f'(ξ) = [f(b)-f(a)]/(b-a) often leads to bounds.
MVT fails if hypotheses aren't met. Always verify: (1) continuity on [a,b], (2) differentiability on (a,b), (3) any additional conditions like f(a)=f(b) for Rolle's.
When g(x) = x, Cauchy's MVT reduces to Lagrange's. Cauchy's handles ratios of function changes, essential for L'Hôpital's rule.
Local extremum + differentiable ⟹ f'(x₀) = 0
f(a) = f(b) ⟹ ∃ξ: f'(ξ) = 0
Derivatives have the Intermediate Value Property
f' > 0 ⟹ f increasing; f' < 0 ⟹ f decreasing
For inequality proofs: identify the function, choose the right interval, apply MVT, then use bounds on f' to conclude.
Always verify: (1) continuity on closed interval, (2) differentiability on open interval. Theorems fail without these!
To prove a function has exactly one root: show existence (IVT), then show uniqueness (Rolle's theorem by contradiction).
Create helper functions like g(x) = f(x) - cx or h(x) = f(x) - kx² to transform problems into forms where Rolle's theorem applies.
In the next module, you will learn about L'Hôpital's Rule for evaluating limits and Taylor's Theorem for polynomial approximations.
Forgetting to check hypotheses
MVT fails if f is not continuous on [a,b] or not differentiable on (a,b).
Confusing local and global extrema
Fermat's theorem only applies to local extrema in the interior.
Thinking f'(x₀) = 0 implies extremum
The converse of Fermat's theorem is false! Consider f(x) = x³ at x = 0.
Misapplying Rolle vs Lagrange
Use Rolle when f(a) = f(b); use Lagrange in general cases.